What value of $n$ will make a triangle contain 560 lattice points?Equilateral triangle whose vertices are lattice points?Number of lattice pointsNumber of lattice points inside a triangle and its areaHow to calculate the number of lattice points in the interior and on the boundary of these figures with vertices as lattice points?Sum of areas of triangles which have corners which are lattice points with 74 lattice points inside.Proving Pick's theoremNumber of lattice points in triangle formed by x-axis, y-axis and given lineOn “small triangles” in a square latticeNumber of lattice points inside a right-angled triangle with one of the points having a rational coordinateequilateral triangle lattice help

Do Iron Man suits sport waste management systems?

Is it a bad idea to plug the other end of ESD strap to wall ground?

How to show a landlord what we have in savings?

What exactly is ineptocracy?

What is the opposite of "eschatology"?

Knowledge-based authentication using Domain-driven Design in C#

Processor speed limited at 0.4 Ghz

GFCI outlets - can they be repaired? Are they really needed at the end of a circuit?

Venezuelan girlfriend wants to travel the USA to be with me. What is the process?

Using "tail" to follow a file without displaying the most recent lines

How to prevent "they're falling in love" trope

Does int main() need a declaration on C++?

Fair gambler's ruin problem intuition

What Exploit Are These User Agents Trying to Use?

Is there a hemisphere-neutral way of specifying a season?

How does a refinance allow a mortgage to be repaid?

Can someone clarify Hamming's notion of important problems in relation to modern academia?

Send out email when Apex Queueable fails and test it

What is the fastest integer factorization to break RSA?

Should I tell management that I intend to leave due to bad software development practices?

Is it possible to create a QR code using text?

Finitely generated matrix groups whose eigenvalues are all algebraic

How to Prove P(a) → ∀x(P(x) ∨ ¬(x = a)) using Natural Deduction

How to remove border from elements in the last row?



What value of $n$ will make a triangle contain 560 lattice points?


Equilateral triangle whose vertices are lattice points?Number of lattice pointsNumber of lattice points inside a triangle and its areaHow to calculate the number of lattice points in the interior and on the boundary of these figures with vertices as lattice points?Sum of areas of triangles which have corners which are lattice points with 74 lattice points inside.Proving Pick's theoremNumber of lattice points in triangle formed by x-axis, y-axis and given lineOn “small triangles” in a square latticeNumber of lattice points inside a right-angled triangle with one of the points having a rational coordinateequilateral triangle lattice help













2












$begingroup$


I recently met a rather hard problem:




A lattice point is a ordered pair where both $x$ and $y$ of $(x, y)$
are both integers. A triangle is forms by of the lattice points $(1, 1)$, $(9, 1)$, and $(9, n)$. For what integer value of $n>0$ are there
exactly 560 lattice points strictly in the interior of the triangle?




My first approach was trying to set an equation using Shoelace and Pick's. I found the area via Shoelace formula in terms of $n$ and then all I needed to do was to set this equal to Pick's theorem. However, I found it extremely hard to count the # of lattice points on the "lines" of the triangle, so that I didn't get very far.



I then tried another approach.
I begin by drawing a diagram:
enter image description here



Since the triangle is a right triangle, I can add a congruent triangle to make a rectangle, with the right angle at $(1, n)$. Since both triangles are congruent, then the entire rectangle has $560cdot 2=1120$ lattice points in the interior. There are 7 possible $x$ coordinates I labeled in the diagram that can serve as valid $x$ coordinated for interior lattice points, so thus, there are $frac11207=160$ possible $y$ coordinates for the interior lattice points.



I have highlighted in red the 3 invalid $y$ coordinates that cannot serve as a $y$ coordinate for an interior lattice point. Thus, we can see that $n-3$(3 for the 3 invalid $y$ coordinates)$=160$. This means that $n=163$. However, this is wrong.



What am I doing wrong? What hole is there in my logic?



Furthermore, could I solve for $n$ using my first method(Shoelace=Pick's and solve)?



Is there any other way to solve this if I can't solve it this way?



Thanks!



Max0815










share|cite|improve this question











$endgroup$
















    2












    $begingroup$


    I recently met a rather hard problem:




    A lattice point is a ordered pair where both $x$ and $y$ of $(x, y)$
    are both integers. A triangle is forms by of the lattice points $(1, 1)$, $(9, 1)$, and $(9, n)$. For what integer value of $n>0$ are there
    exactly 560 lattice points strictly in the interior of the triangle?




    My first approach was trying to set an equation using Shoelace and Pick's. I found the area via Shoelace formula in terms of $n$ and then all I needed to do was to set this equal to Pick's theorem. However, I found it extremely hard to count the # of lattice points on the "lines" of the triangle, so that I didn't get very far.



    I then tried another approach.
    I begin by drawing a diagram:
    enter image description here



    Since the triangle is a right triangle, I can add a congruent triangle to make a rectangle, with the right angle at $(1, n)$. Since both triangles are congruent, then the entire rectangle has $560cdot 2=1120$ lattice points in the interior. There are 7 possible $x$ coordinates I labeled in the diagram that can serve as valid $x$ coordinated for interior lattice points, so thus, there are $frac11207=160$ possible $y$ coordinates for the interior lattice points.



    I have highlighted in red the 3 invalid $y$ coordinates that cannot serve as a $y$ coordinate for an interior lattice point. Thus, we can see that $n-3$(3 for the 3 invalid $y$ coordinates)$=160$. This means that $n=163$. However, this is wrong.



    What am I doing wrong? What hole is there in my logic?



    Furthermore, could I solve for $n$ using my first method(Shoelace=Pick's and solve)?



    Is there any other way to solve this if I can't solve it this way?



    Thanks!



    Max0815










    share|cite|improve this question











    $endgroup$














      2












      2








      2





      $begingroup$


      I recently met a rather hard problem:




      A lattice point is a ordered pair where both $x$ and $y$ of $(x, y)$
      are both integers. A triangle is forms by of the lattice points $(1, 1)$, $(9, 1)$, and $(9, n)$. For what integer value of $n>0$ are there
      exactly 560 lattice points strictly in the interior of the triangle?




      My first approach was trying to set an equation using Shoelace and Pick's. I found the area via Shoelace formula in terms of $n$ and then all I needed to do was to set this equal to Pick's theorem. However, I found it extremely hard to count the # of lattice points on the "lines" of the triangle, so that I didn't get very far.



      I then tried another approach.
      I begin by drawing a diagram:
      enter image description here



      Since the triangle is a right triangle, I can add a congruent triangle to make a rectangle, with the right angle at $(1, n)$. Since both triangles are congruent, then the entire rectangle has $560cdot 2=1120$ lattice points in the interior. There are 7 possible $x$ coordinates I labeled in the diagram that can serve as valid $x$ coordinated for interior lattice points, so thus, there are $frac11207=160$ possible $y$ coordinates for the interior lattice points.



      I have highlighted in red the 3 invalid $y$ coordinates that cannot serve as a $y$ coordinate for an interior lattice point. Thus, we can see that $n-3$(3 for the 3 invalid $y$ coordinates)$=160$. This means that $n=163$. However, this is wrong.



      What am I doing wrong? What hole is there in my logic?



      Furthermore, could I solve for $n$ using my first method(Shoelace=Pick's and solve)?



      Is there any other way to solve this if I can't solve it this way?



      Thanks!



      Max0815










      share|cite|improve this question











      $endgroup$




      I recently met a rather hard problem:




      A lattice point is a ordered pair where both $x$ and $y$ of $(x, y)$
      are both integers. A triangle is forms by of the lattice points $(1, 1)$, $(9, 1)$, and $(9, n)$. For what integer value of $n>0$ are there
      exactly 560 lattice points strictly in the interior of the triangle?




      My first approach was trying to set an equation using Shoelace and Pick's. I found the area via Shoelace formula in terms of $n$ and then all I needed to do was to set this equal to Pick's theorem. However, I found it extremely hard to count the # of lattice points on the "lines" of the triangle, so that I didn't get very far.



      I then tried another approach.
      I begin by drawing a diagram:
      enter image description here



      Since the triangle is a right triangle, I can add a congruent triangle to make a rectangle, with the right angle at $(1, n)$. Since both triangles are congruent, then the entire rectangle has $560cdot 2=1120$ lattice points in the interior. There are 7 possible $x$ coordinates I labeled in the diagram that can serve as valid $x$ coordinated for interior lattice points, so thus, there are $frac11207=160$ possible $y$ coordinates for the interior lattice points.



      I have highlighted in red the 3 invalid $y$ coordinates that cannot serve as a $y$ coordinate for an interior lattice point. Thus, we can see that $n-3$(3 for the 3 invalid $y$ coordinates)$=160$. This means that $n=163$. However, this is wrong.



      What am I doing wrong? What hole is there in my logic?



      Furthermore, could I solve for $n$ using my first method(Shoelace=Pick's and solve)?



      Is there any other way to solve this if I can't solve it this way?



      Thanks!



      Max0815







      geometry logic






      share|cite|improve this question















      share|cite|improve this question













      share|cite|improve this question




      share|cite|improve this question








      edited Mar 20 at 19:43







      Max0815

















      asked Mar 20 at 19:37









      Max0815Max0815

      81418




      81418




















          2 Answers
          2






          active

          oldest

          votes


















          1












          $begingroup$

          Your error is that the lattice points on the diagonal from $(1,1)$ to $(9,n)$ (excluding the endpoints) are interior points of the rectangle, but are not counted in the $2times560$ points.



          The main takeaway is that you will have to deal with that pesky diagonal one way or another. The triangle being right-angled with coordinate-parallel sides allows both approaches: The shoelace/Pick approach is more general, the second is easier, especially if you don't know about the Pick formula.



          I'll give some hints for the first approach:



          1) From the 3 sides of the triangle, 2 should be trivial by now, you did them for the second approach!



          2) That leads to the diagonal, which represents a displacement of $(8,n-1)$, or if you look at the linear function on which that diagonal lies, it is $y=fracn-18(x-1)+1$



          3) Any lattice point in the interior of that diagonal is characerized by 2 conditions:



          a) $2le x le 8$, and



          b)$fracn-18(x-1)$ must be an integer.



          4) So the key question becomes: For which/how many $x'=x-1$ satisfying $1 le x' le 7$ is $fracn-18x'$ an integer?



          5) The answer depends of course on the value of $fracn-18$. If that is already an integer (so $8|(n-1)$), every $1 le x' le 7$ works. OTOH, if $n-1$ is odd, then $x'$ would have to be divisible by $8$, which it can't be in that range.



          6) Can you find the remaining cases? Can you find the respective number of solution $x'$?
          If you can't, try $n-1=2,4,6$ maybe you see something!



          7) The previous parts of the problem should all be 'simple' formulas of $n$. If you can solve 6), you have several cases now for the part of the number of lattice points on the diagonal. Solve them each, then check if the solution (if one exists) matches the condition (that means if you use the formula that applies when $8|(n-1)$ and get a solution $n=165$, then it can't be correct, as $8$ does not divide $164$.






          share|cite|improve this answer









          $endgroup$












          • $begingroup$
            Can you also help me on my second approach?
            $endgroup$
            – Max0815
            Mar 22 at 1:37










          • $begingroup$
            The help is basically the same. As I wrote, the error you made is that you did not account for the lattice points on the $(1,1)-(9,n)$ diagonal. I tried to jumpstart that calculation with my answer. If you have the result, you can use it for either approach.
            $endgroup$
            – Ingix
            Mar 22 at 10:32










          • $begingroup$
            Ok. Got it. Thank you.
            $endgroup$
            – Max0815
            Mar 23 at 21:41


















          0












          $begingroup$

          enter image description here



          Since the triangle is a right triangle, I can add a congruent triangle to make a rectangle, with the right angle at $(1, n)$. Since both triangles are congruent, then the entire rectangle has $560cdot 2=1120$ lattice points in the interior. There are 7 possible $x$ coordinates I labeled in the diagram that can serve as valid $x$ coordinated for interior lattice points, so thus, there are $frac11207=160$ possible $y$ coordinates for the interior lattice points.



          I have highlighted in red the 3 invalid $y$ coordinates that cannot serve as a $y$ coordinate for an interior lattice point. However, when $y=0$, this line has lattice points that will not ever be in the triangle bound by $(9, n)$, and the other two points in the problem, so only the top and middles lines highlighted in red serve as invalid. Thus, we can see that $n-2=160$. This means that $n=boxed162$.






          share|cite|improve this answer









          $endgroup$













            Your Answer





            StackExchange.ifUsing("editor", function ()
            return StackExchange.using("mathjaxEditing", function ()
            StackExchange.MarkdownEditor.creationCallbacks.add(function (editor, postfix)
            StackExchange.mathjaxEditing.prepareWmdForMathJax(editor, postfix, [["$", "$"], ["\\(","\\)"]]);
            );
            );
            , "mathjax-editing");

            StackExchange.ready(function()
            var channelOptions =
            tags: "".split(" "),
            id: "69"
            ;
            initTagRenderer("".split(" "), "".split(" "), channelOptions);

            StackExchange.using("externalEditor", function()
            // Have to fire editor after snippets, if snippets enabled
            if (StackExchange.settings.snippets.snippetsEnabled)
            StackExchange.using("snippets", function()
            createEditor();
            );

            else
            createEditor();

            );

            function createEditor()
            StackExchange.prepareEditor(
            heartbeatType: 'answer',
            autoActivateHeartbeat: false,
            convertImagesToLinks: true,
            noModals: true,
            showLowRepImageUploadWarning: true,
            reputationToPostImages: 10,
            bindNavPrevention: true,
            postfix: "",
            imageUploader:
            brandingHtml: "Powered by u003ca class="icon-imgur-white" href="https://imgur.com/"u003eu003c/au003e",
            contentPolicyHtml: "User contributions licensed under u003ca href="https://creativecommons.org/licenses/by-sa/3.0/"u003ecc by-sa 3.0 with attribution requiredu003c/au003e u003ca href="https://stackoverflow.com/legal/content-policy"u003e(content policy)u003c/au003e",
            allowUrls: true
            ,
            noCode: true, onDemand: true,
            discardSelector: ".discard-answer"
            ,immediatelyShowMarkdownHelp:true
            );



            );













            draft saved

            draft discarded


















            StackExchange.ready(
            function ()
            StackExchange.openid.initPostLogin('.new-post-login', 'https%3a%2f%2fmath.stackexchange.com%2fquestions%2f3155914%2fwhat-value-of-n-will-make-a-triangle-contain-560-lattice-points%23new-answer', 'question_page');

            );

            Post as a guest















            Required, but never shown

























            2 Answers
            2






            active

            oldest

            votes








            2 Answers
            2






            active

            oldest

            votes









            active

            oldest

            votes






            active

            oldest

            votes









            1












            $begingroup$

            Your error is that the lattice points on the diagonal from $(1,1)$ to $(9,n)$ (excluding the endpoints) are interior points of the rectangle, but are not counted in the $2times560$ points.



            The main takeaway is that you will have to deal with that pesky diagonal one way or another. The triangle being right-angled with coordinate-parallel sides allows both approaches: The shoelace/Pick approach is more general, the second is easier, especially if you don't know about the Pick formula.



            I'll give some hints for the first approach:



            1) From the 3 sides of the triangle, 2 should be trivial by now, you did them for the second approach!



            2) That leads to the diagonal, which represents a displacement of $(8,n-1)$, or if you look at the linear function on which that diagonal lies, it is $y=fracn-18(x-1)+1$



            3) Any lattice point in the interior of that diagonal is characerized by 2 conditions:



            a) $2le x le 8$, and



            b)$fracn-18(x-1)$ must be an integer.



            4) So the key question becomes: For which/how many $x'=x-1$ satisfying $1 le x' le 7$ is $fracn-18x'$ an integer?



            5) The answer depends of course on the value of $fracn-18$. If that is already an integer (so $8|(n-1)$), every $1 le x' le 7$ works. OTOH, if $n-1$ is odd, then $x'$ would have to be divisible by $8$, which it can't be in that range.



            6) Can you find the remaining cases? Can you find the respective number of solution $x'$?
            If you can't, try $n-1=2,4,6$ maybe you see something!



            7) The previous parts of the problem should all be 'simple' formulas of $n$. If you can solve 6), you have several cases now for the part of the number of lattice points on the diagonal. Solve them each, then check if the solution (if one exists) matches the condition (that means if you use the formula that applies when $8|(n-1)$ and get a solution $n=165$, then it can't be correct, as $8$ does not divide $164$.






            share|cite|improve this answer









            $endgroup$












            • $begingroup$
              Can you also help me on my second approach?
              $endgroup$
              – Max0815
              Mar 22 at 1:37










            • $begingroup$
              The help is basically the same. As I wrote, the error you made is that you did not account for the lattice points on the $(1,1)-(9,n)$ diagonal. I tried to jumpstart that calculation with my answer. If you have the result, you can use it for either approach.
              $endgroup$
              – Ingix
              Mar 22 at 10:32










            • $begingroup$
              Ok. Got it. Thank you.
              $endgroup$
              – Max0815
              Mar 23 at 21:41















            1












            $begingroup$

            Your error is that the lattice points on the diagonal from $(1,1)$ to $(9,n)$ (excluding the endpoints) are interior points of the rectangle, but are not counted in the $2times560$ points.



            The main takeaway is that you will have to deal with that pesky diagonal one way or another. The triangle being right-angled with coordinate-parallel sides allows both approaches: The shoelace/Pick approach is more general, the second is easier, especially if you don't know about the Pick formula.



            I'll give some hints for the first approach:



            1) From the 3 sides of the triangle, 2 should be trivial by now, you did them for the second approach!



            2) That leads to the diagonal, which represents a displacement of $(8,n-1)$, or if you look at the linear function on which that diagonal lies, it is $y=fracn-18(x-1)+1$



            3) Any lattice point in the interior of that diagonal is characerized by 2 conditions:



            a) $2le x le 8$, and



            b)$fracn-18(x-1)$ must be an integer.



            4) So the key question becomes: For which/how many $x'=x-1$ satisfying $1 le x' le 7$ is $fracn-18x'$ an integer?



            5) The answer depends of course on the value of $fracn-18$. If that is already an integer (so $8|(n-1)$), every $1 le x' le 7$ works. OTOH, if $n-1$ is odd, then $x'$ would have to be divisible by $8$, which it can't be in that range.



            6) Can you find the remaining cases? Can you find the respective number of solution $x'$?
            If you can't, try $n-1=2,4,6$ maybe you see something!



            7) The previous parts of the problem should all be 'simple' formulas of $n$. If you can solve 6), you have several cases now for the part of the number of lattice points on the diagonal. Solve them each, then check if the solution (if one exists) matches the condition (that means if you use the formula that applies when $8|(n-1)$ and get a solution $n=165$, then it can't be correct, as $8$ does not divide $164$.






            share|cite|improve this answer









            $endgroup$












            • $begingroup$
              Can you also help me on my second approach?
              $endgroup$
              – Max0815
              Mar 22 at 1:37










            • $begingroup$
              The help is basically the same. As I wrote, the error you made is that you did not account for the lattice points on the $(1,1)-(9,n)$ diagonal. I tried to jumpstart that calculation with my answer. If you have the result, you can use it for either approach.
              $endgroup$
              – Ingix
              Mar 22 at 10:32










            • $begingroup$
              Ok. Got it. Thank you.
              $endgroup$
              – Max0815
              Mar 23 at 21:41













            1












            1








            1





            $begingroup$

            Your error is that the lattice points on the diagonal from $(1,1)$ to $(9,n)$ (excluding the endpoints) are interior points of the rectangle, but are not counted in the $2times560$ points.



            The main takeaway is that you will have to deal with that pesky diagonal one way or another. The triangle being right-angled with coordinate-parallel sides allows both approaches: The shoelace/Pick approach is more general, the second is easier, especially if you don't know about the Pick formula.



            I'll give some hints for the first approach:



            1) From the 3 sides of the triangle, 2 should be trivial by now, you did them for the second approach!



            2) That leads to the diagonal, which represents a displacement of $(8,n-1)$, or if you look at the linear function on which that diagonal lies, it is $y=fracn-18(x-1)+1$



            3) Any lattice point in the interior of that diagonal is characerized by 2 conditions:



            a) $2le x le 8$, and



            b)$fracn-18(x-1)$ must be an integer.



            4) So the key question becomes: For which/how many $x'=x-1$ satisfying $1 le x' le 7$ is $fracn-18x'$ an integer?



            5) The answer depends of course on the value of $fracn-18$. If that is already an integer (so $8|(n-1)$), every $1 le x' le 7$ works. OTOH, if $n-1$ is odd, then $x'$ would have to be divisible by $8$, which it can't be in that range.



            6) Can you find the remaining cases? Can you find the respective number of solution $x'$?
            If you can't, try $n-1=2,4,6$ maybe you see something!



            7) The previous parts of the problem should all be 'simple' formulas of $n$. If you can solve 6), you have several cases now for the part of the number of lattice points on the diagonal. Solve them each, then check if the solution (if one exists) matches the condition (that means if you use the formula that applies when $8|(n-1)$ and get a solution $n=165$, then it can't be correct, as $8$ does not divide $164$.






            share|cite|improve this answer









            $endgroup$



            Your error is that the lattice points on the diagonal from $(1,1)$ to $(9,n)$ (excluding the endpoints) are interior points of the rectangle, but are not counted in the $2times560$ points.



            The main takeaway is that you will have to deal with that pesky diagonal one way or another. The triangle being right-angled with coordinate-parallel sides allows both approaches: The shoelace/Pick approach is more general, the second is easier, especially if you don't know about the Pick formula.



            I'll give some hints for the first approach:



            1) From the 3 sides of the triangle, 2 should be trivial by now, you did them for the second approach!



            2) That leads to the diagonal, which represents a displacement of $(8,n-1)$, or if you look at the linear function on which that diagonal lies, it is $y=fracn-18(x-1)+1$



            3) Any lattice point in the interior of that diagonal is characerized by 2 conditions:



            a) $2le x le 8$, and



            b)$fracn-18(x-1)$ must be an integer.



            4) So the key question becomes: For which/how many $x'=x-1$ satisfying $1 le x' le 7$ is $fracn-18x'$ an integer?



            5) The answer depends of course on the value of $fracn-18$. If that is already an integer (so $8|(n-1)$), every $1 le x' le 7$ works. OTOH, if $n-1$ is odd, then $x'$ would have to be divisible by $8$, which it can't be in that range.



            6) Can you find the remaining cases? Can you find the respective number of solution $x'$?
            If you can't, try $n-1=2,4,6$ maybe you see something!



            7) The previous parts of the problem should all be 'simple' formulas of $n$. If you can solve 6), you have several cases now for the part of the number of lattice points on the diagonal. Solve them each, then check if the solution (if one exists) matches the condition (that means if you use the formula that applies when $8|(n-1)$ and get a solution $n=165$, then it can't be correct, as $8$ does not divide $164$.







            share|cite|improve this answer












            share|cite|improve this answer



            share|cite|improve this answer










            answered Mar 20 at 20:19









            IngixIngix

            5,087159




            5,087159











            • $begingroup$
              Can you also help me on my second approach?
              $endgroup$
              – Max0815
              Mar 22 at 1:37










            • $begingroup$
              The help is basically the same. As I wrote, the error you made is that you did not account for the lattice points on the $(1,1)-(9,n)$ diagonal. I tried to jumpstart that calculation with my answer. If you have the result, you can use it for either approach.
              $endgroup$
              – Ingix
              Mar 22 at 10:32










            • $begingroup$
              Ok. Got it. Thank you.
              $endgroup$
              – Max0815
              Mar 23 at 21:41
















            • $begingroup$
              Can you also help me on my second approach?
              $endgroup$
              – Max0815
              Mar 22 at 1:37










            • $begingroup$
              The help is basically the same. As I wrote, the error you made is that you did not account for the lattice points on the $(1,1)-(9,n)$ diagonal. I tried to jumpstart that calculation with my answer. If you have the result, you can use it for either approach.
              $endgroup$
              – Ingix
              Mar 22 at 10:32










            • $begingroup$
              Ok. Got it. Thank you.
              $endgroup$
              – Max0815
              Mar 23 at 21:41















            $begingroup$
            Can you also help me on my second approach?
            $endgroup$
            – Max0815
            Mar 22 at 1:37




            $begingroup$
            Can you also help me on my second approach?
            $endgroup$
            – Max0815
            Mar 22 at 1:37












            $begingroup$
            The help is basically the same. As I wrote, the error you made is that you did not account for the lattice points on the $(1,1)-(9,n)$ diagonal. I tried to jumpstart that calculation with my answer. If you have the result, you can use it for either approach.
            $endgroup$
            – Ingix
            Mar 22 at 10:32




            $begingroup$
            The help is basically the same. As I wrote, the error you made is that you did not account for the lattice points on the $(1,1)-(9,n)$ diagonal. I tried to jumpstart that calculation with my answer. If you have the result, you can use it for either approach.
            $endgroup$
            – Ingix
            Mar 22 at 10:32












            $begingroup$
            Ok. Got it. Thank you.
            $endgroup$
            – Max0815
            Mar 23 at 21:41




            $begingroup$
            Ok. Got it. Thank you.
            $endgroup$
            – Max0815
            Mar 23 at 21:41











            0












            $begingroup$

            enter image description here



            Since the triangle is a right triangle, I can add a congruent triangle to make a rectangle, with the right angle at $(1, n)$. Since both triangles are congruent, then the entire rectangle has $560cdot 2=1120$ lattice points in the interior. There are 7 possible $x$ coordinates I labeled in the diagram that can serve as valid $x$ coordinated for interior lattice points, so thus, there are $frac11207=160$ possible $y$ coordinates for the interior lattice points.



            I have highlighted in red the 3 invalid $y$ coordinates that cannot serve as a $y$ coordinate for an interior lattice point. However, when $y=0$, this line has lattice points that will not ever be in the triangle bound by $(9, n)$, and the other two points in the problem, so only the top and middles lines highlighted in red serve as invalid. Thus, we can see that $n-2=160$. This means that $n=boxed162$.






            share|cite|improve this answer









            $endgroup$

















              0












              $begingroup$

              enter image description here



              Since the triangle is a right triangle, I can add a congruent triangle to make a rectangle, with the right angle at $(1, n)$. Since both triangles are congruent, then the entire rectangle has $560cdot 2=1120$ lattice points in the interior. There are 7 possible $x$ coordinates I labeled in the diagram that can serve as valid $x$ coordinated for interior lattice points, so thus, there are $frac11207=160$ possible $y$ coordinates for the interior lattice points.



              I have highlighted in red the 3 invalid $y$ coordinates that cannot serve as a $y$ coordinate for an interior lattice point. However, when $y=0$, this line has lattice points that will not ever be in the triangle bound by $(9, n)$, and the other two points in the problem, so only the top and middles lines highlighted in red serve as invalid. Thus, we can see that $n-2=160$. This means that $n=boxed162$.






              share|cite|improve this answer









              $endgroup$















                0












                0








                0





                $begingroup$

                enter image description here



                Since the triangle is a right triangle, I can add a congruent triangle to make a rectangle, with the right angle at $(1, n)$. Since both triangles are congruent, then the entire rectangle has $560cdot 2=1120$ lattice points in the interior. There are 7 possible $x$ coordinates I labeled in the diagram that can serve as valid $x$ coordinated for interior lattice points, so thus, there are $frac11207=160$ possible $y$ coordinates for the interior lattice points.



                I have highlighted in red the 3 invalid $y$ coordinates that cannot serve as a $y$ coordinate for an interior lattice point. However, when $y=0$, this line has lattice points that will not ever be in the triangle bound by $(9, n)$, and the other two points in the problem, so only the top and middles lines highlighted in red serve as invalid. Thus, we can see that $n-2=160$. This means that $n=boxed162$.






                share|cite|improve this answer









                $endgroup$



                enter image description here



                Since the triangle is a right triangle, I can add a congruent triangle to make a rectangle, with the right angle at $(1, n)$. Since both triangles are congruent, then the entire rectangle has $560cdot 2=1120$ lattice points in the interior. There are 7 possible $x$ coordinates I labeled in the diagram that can serve as valid $x$ coordinated for interior lattice points, so thus, there are $frac11207=160$ possible $y$ coordinates for the interior lattice points.



                I have highlighted in red the 3 invalid $y$ coordinates that cannot serve as a $y$ coordinate for an interior lattice point. However, when $y=0$, this line has lattice points that will not ever be in the triangle bound by $(9, n)$, and the other two points in the problem, so only the top and middles lines highlighted in red serve as invalid. Thus, we can see that $n-2=160$. This means that $n=boxed162$.







                share|cite|improve this answer












                share|cite|improve this answer



                share|cite|improve this answer










                answered Mar 23 at 21:44









                Max0815Max0815

                81418




                81418



























                    draft saved

                    draft discarded
















































                    Thanks for contributing an answer to Mathematics Stack Exchange!


                    • Please be sure to answer the question. Provide details and share your research!

                    But avoid


                    • Asking for help, clarification, or responding to other answers.

                    • Making statements based on opinion; back them up with references or personal experience.

                    Use MathJax to format equations. MathJax reference.


                    To learn more, see our tips on writing great answers.




                    draft saved


                    draft discarded














                    StackExchange.ready(
                    function ()
                    StackExchange.openid.initPostLogin('.new-post-login', 'https%3a%2f%2fmath.stackexchange.com%2fquestions%2f3155914%2fwhat-value-of-n-will-make-a-triangle-contain-560-lattice-points%23new-answer', 'question_page');

                    );

                    Post as a guest















                    Required, but never shown





















































                    Required, but never shown














                    Required, but never shown












                    Required, but never shown







                    Required, but never shown

































                    Required, but never shown














                    Required, but never shown












                    Required, but never shown







                    Required, but never shown







                    Popular posts from this blog

                    Lowndes Grove History Architecture References Navigation menu32°48′6″N 79°57′58″W / 32.80167°N 79.96611°W / 32.80167; -79.9661132°48′6″N 79°57′58″W / 32.80167°N 79.96611°W / 32.80167; -79.9661178002500"National Register Information System"Historic houses of South Carolina"Lowndes Grove""+32° 48' 6.00", −79° 57' 58.00""Lowndes Grove, Charleston County (260 St. Margaret St., Charleston)""Lowndes Grove"The Charleston ExpositionIt Happened in South Carolina"Lowndes Grove (House), Saint Margaret Street & Sixth Avenue, Charleston, Charleston County, SC(Photographs)"Plantations of the Carolina Low Countrye

                    random experiment with two different functions on unit interval Announcing the arrival of Valued Associate #679: Cesar Manara Planned maintenance scheduled April 23, 2019 at 00:00UTC (8:00pm US/Eastern)Random variable and probability space notionsRandom Walk with EdgesFinding functions where the increase over a random interval is Poisson distributedNumber of days until dayCan an observed event in fact be of zero probability?Unit random processmodels of coins and uniform distributionHow to get the number of successes given $n$ trials , probability $P$ and a random variable $X$Absorbing Markov chain in a computer. Is “almost every” turned into always convergence in computer executions?Stopped random walk is not uniformly integrable

                    How should I support this large drywall patch? Planned maintenance scheduled April 23, 2019 at 00:00UTC (8:00pm US/Eastern) Announcing the arrival of Valued Associate #679: Cesar Manara Unicorn Meta Zoo #1: Why another podcast?How do I cover large gaps in drywall?How do I keep drywall around a patch from crumbling?Can I glue a second layer of drywall?How to patch long strip on drywall?Large drywall patch: how to avoid bulging seams?Drywall Mesh Patch vs. Bulge? To remove or not to remove?How to fix this drywall job?Prep drywall before backsplashWhat's the best way to fix this horrible drywall patch job?Drywall patching using 3M Patch Plus Primer